University Calculus: Early Transcendentals (3rd Edition)

Published by Pearson
ISBN 10: 0321999584
ISBN 13: 978-0-32199-958-0

Chapter 9 - Section 9.10 - The Binomial Series and Applications of Taylor Series - Exercises - Page 549: 20

Answer

$\approx 0.0996676643$ with an error of about $4.6 \times 10^{-12}$

Work Step by Step

We integrate the integral with respect to $ x $ taking a lower limit as $0$ and upper limit as $0.1$. $\int_0^{0.1} e^{-x^2} dx=\int_0^{0.1} [1-x^2+\dfrac{x^4}{2!}-\dfrac{x^{6}}{3!}+...] dx $ or, $=0.1-\dfrac{(0.1)^3}{3 }+\dfrac{(0.1)^5}{5 \cdot 2!}-...$ But $\dfrac{(0.1)^9}{9 \cdot 4!}\approx 4.6 \times 10^{-12}$ and the proceeding term is greater than $10^{-8}$. so, the first three terms of the series would give the accuracy. $\int_0^{0.1}e^{-x^2} dx \approx 0.0996676643$ with an error of about $4.6 \times 10^{-12}$
Update this answer!

You can help us out by revising, improving and updating this answer.

Update this answer

After you claim an answer you’ll have 24 hours to send in a draft. An editor will review the submission and either publish your submission or provide feedback.